0 Daumen
297 Aufrufe

Aufgabe:dimK(W1 + W2 + W3) = dimK W1 + dimK W2 + dimK W3
− dimK(W1 ∩ W2) − dimK(W2 ∩ W3)
− dimK(W1 ∩ W3) + dimK(W1 ∩ W2 ∩ W3).
Ist das wahr?


Problem/Ansatz: Ich weiß, dass es dort einen Widerspruch gibt, weiß aber nicht wie ich es beweisen kann.

Avatar von

1 Antwort

0 Daumen

Hallo,

diese Frage ist ein Klassiker im Netz. Nimm im \(\mathbb{R}^2\) 3 paarweise verschiedene Geraden durch den Nullpunkt.

Gußr Mathhilf

Avatar von 13 k

Ein anderes Problem?

Stell deine Frage

Willkommen bei der Mathelounge! Stell deine Frage einfach und kostenlos

x
Made by a lovely community